LSAT and Law School Admissions Forum

Get expert LSAT preparation and law school admissions advice from PowerScore Test Preparation.

 Shannon Parker
PowerScore Staff
  • PowerScore Staff
  • Posts: 147
  • Joined: Jun 08, 2016
|
#42427
Etsevdos wrote:If the argument said: "Since all sides of an imp. story should be covered and no newspr covers all important sides. some important stories would not be adequately covered if only 1 newspaper" would that be correct?
Etsevdos,

You are on the right track. If the stimulus read, "Everyone should have access to more than one newspaper, for there are at least two sides to every story. Since all sides of an important story should be covered and no newspaper covers all sides of every important story, some important stories would not be adequately covered if there were only one newspaper." The flaw described in answer choice A would be removed, and it would be logically sound.

Hope this helps.
 Khodi7531
  • Posts: 116
  • Joined: Mar 14, 2018
|
#45738
I had found the jump in language in this question from the conclusion of "should have access to more than one newspaper" to it's reasoning that talks about "important stories".

I think my issue of choosing B - and leaving C and A as contenders as well - is rooted in my recently diagnosed mistake. Which is, what is the question actually asking me to do?

"describe a flaw in the reasoning" isn't necessarily weakening it, right? I know the answer can still do just that by pointing to an issue, but i'm not working to shi* on the authors reasoning. I'm more so pointing out the issue from how you can't reach this conclusion from the reasoning presented.

B) "the argument overlooks the possibility" is more-so weakening the conclusion... while A) states, "argument confused the inability.." to point at it's illogical jumps.


Is this part of what I should be looking at to differentiate tempting answers with others?
 Adam Tyson
PowerScore Staff
  • PowerScore Staff
  • Posts: 5153
  • Joined: Apr 14, 2011
|
#49562
Weaken and Flaw are very closely related questions, khodi, and it can be easy to mix them up. Think of it this way - a Flaw answer points out what went wrong, while a weaken answer takes advantage of that problem. "Describes a flaw" is definitely about the flaw, rather than weakening.

"Failed to consider" is commonly a Flaw description, not a Weaken. So is "overlooks the possibility" and "the argument confuses" something. All of these are just abstract descriptions of a problem in the argument. To weaken, you go a step further and actually say "it could be that (the thing you didn't consider) is true." Weaken stems tend to ask which answer, if true, will do most to undermine or weaken the argument or call into question the conclusion. Not just say what's wrong, but make you doubt the conclusion. It's more than a description, it's an attack!

The conclusion here is that everyone should have access to more than one newspaper. The evidence is that one newspaper cannot cover all sides of every story, and it's important to cover all sides of the important stories. I've added emphasis here to show the gap in the argument, which is where the flaw lives. Your prephrase might be something like "the author failed to consider that one newspaper might cover all sides of the important stories even if they don't cover all the stories that way."
 menkenj
  • Posts: 116
  • Joined: Dec 02, 2020
|
#82727
Why is B wrong? I am having difficulty understanding.
User avatar
 daydreamingsamosa
  • Posts: 4
  • Joined: Dec 31, 2020
|
#83084
I get why this argument is wrong. It's wrong because just because you can't cover all sides of every story, it's still possible to cover all sides of important stories and not cover all sides of unimportant ones. BUT I was a bit hesitant with AC (A) because by saying "inability to cover all sides of any important story" it's saying that no important story will get full coverage of all sides. But that's a bit more extreme than what the argument is saying. The argument only said SOME important stories wouldn't be adequately covered not that no important story will get full coverage. The stimulus leaves open the possibility that some important stories don't get covered adequately while some do, but AC (A) says something more extreme. Please help?
User avatar
 KelseyWoods
PowerScore Staff
  • PowerScore Staff
  • Posts: 1079
  • Joined: Jun 26, 2013
|
#83221
Hi menkenj & daydreamingsamosa!

menkenj: Answer choice (B) is incorrect because the author does not overlook the possibility that two newspapers could provide the same incomplete coverage of the same important stories. The author says that everyone should have access to "more than one" newspaper but nowhere does the author say that if you have access to two newspapers you'll get complete coverage. Basically the author is saying that more than one newspaper is necessary to get complete coverage but does not say that more than one newspaper is sufficient to get complete coverage and also doesn't say that specifically two newspapers would provide complete coverage.

daydreamingsamosa: The wording here is definitely a little bit tricky! But the phrase "inability to cover all sides of any important story" does not necessarily mean that no important story will get coverage of all sides. On the LSAT, any and every are often used somewhat interchangeably--grammatically speaking, there are slight differences between these terms and they can be used differently in different contexts. But, for example, "any" and "every" are both sufficient indicator terms. So you could interpret it as "inability to cover all sides of every important story"--meaning that there will at least be some important stories that do not get coverage of all sides. You could also interpret "any" to mean "one or more." In that case it would read "inability to cover all sides of one or more important stories." This, again, would mean that there will at least be some important stories that do not get coverage of all sides but does not necessarily mean that no important story will get full coverage.

Hope this helps!

Best,
Kelsey
 menkenj
  • Posts: 116
  • Joined: Dec 02, 2020
|
#83437
KelseyWoods wrote: Wed Jan 13, 2021 3:59 pm Hi menkenj & daydreamingsamosa!

menkenj: Answer choice (B) is incorrect because the author does not overlook the possibility that two newspapers could provide the same incomplete coverage of the same important stories. The author says that everyone should have access to "more than one" newspaper but nowhere does the author say that if you have access to two newspapers you'll get complete coverage. Basically the author is saying that more than one newspaper is necessary to get complete coverage but does not say that more than one newspaper is sufficient to get complete coverage and also doesn't say that specifically two newspapers would provide complete coverage.

daydreamingsamosa: The wording here is definitely a little bit tricky! But the phrase "inability to cover all sides of any important story" does not necessarily mean that no important story will get coverage of all sides. On the LSAT, any and every are often used somewhat interchangeably--grammatically speaking, there are slight differences between these terms and they can be used differently in different contexts. But, for example, "any" and "every" are both sufficient indicator terms. So you could interpret it as "inability to cover all sides of every important story"--meaning that there will at least be some important stories that do not get coverage of all sides. You could also interpret "any" to mean "one or more." In that case it would read "inability to cover all sides of one or more important stories." This, again, would mean that there will at least be some important stories that do not get coverage of all sides but does not necessarily mean that no important story will get full coverage.

Hope this helps!

Best,
Kelsey

Hi Kelsey,

Thanks for your response. I keep coming back to this question to make sure I understand it. I think I'm nearly there.

The argument seems to center around the hypothetical of what would happen if there were only one newspaper. So then (b) is wrong because gaining an incomplete picture of important stories with two newspapers is irrelevant.

The flaw is that the author shifts between important stories and all stories in the premise immediately preceding the conclusion. Presumably no newspaper could adequately cover all sides of every one of its stories and still adequately cover every one of its important stories. Because the author addresses every story instead of every important story in this premise, the conclusion does not follow so there lies the flaw.

Is this an accurate evaluation?
User avatar
 KelseyWoods
PowerScore Staff
  • PowerScore Staff
  • Posts: 1079
  • Joined: Jun 26, 2013
|
#84146
Hi menkenj!

I think you've got it! Even if a newspaper does not cover all sides of every one of its stories, it still might cover all sides of every one of its important stories. And, yes, answer choice (B) is incorrect because the author does not say that having two newspapers is enough to provide complete coverage of all important stories. Rather, the author just says that having one newspaper is not enough to adequately cover the important stories.

Good job!

Best,
Kelsey
User avatar
 Christmaspuppy
  • Posts: 24
  • Joined: Dec 31, 2021
|
#94457
Hi!This one really tripped me up. I can't understand the explanation between "important " to "all". The argument said no newspaper could cover all sides of every one of its stories. Then, if it's a story of this newspaper, it can't be covered of all sides. So, I think there's no way "the important stories" are not a subset of "every story". Could someone help me with this? I am really having trouble with understanding this.
Thank you!
User avatar
 Christmaspuppy
  • Posts: 24
  • Joined: Dec 31, 2021
|
#94497
Christmaspuppy wrote: Thu Mar 24, 2022 9:27 pm Hi!This one really tripped me up. I can't understand the explanation between "important " to "all". The argument said no newspaper could cover all sides of every one of its stories. Then, if it's a story of this newspaper, it can't be covered of all sides. So, I think there's no way "the important stories" are not a subset of "every story". Could someone help me with this? I am really having trouble with understanding this.
Thank you!
Hi again! As you can see, I was having a hard time to figure out why “the important stories” could be out of “every story”. After a few days, something just clicked. I don’t know whether I could interpret it this way. I am thinking the “every story” in the argument supposed to mean “all of its stories” but not”each of its stories”. This kinda make sense to me. Hope I can get a reply this time😸

Get the most out of your LSAT Prep Plus subscription.

Analyze and track your performance with our Testing and Analytics Package.